University Calculus: Early Transcendentals (3rd Edition)

Published by Pearson
ISBN 10: 0321999584
ISBN 13: 978-0-32199-958-0

Chapter 14 - Section 14.4 - Double Integrals in Polar Form - Exercises - Page 778: 5

Answer

$0 \leq \theta \leq \dfrac{\pi}{6}$ and $1\leq r\leq 2 \sqrt 3 \sec \theta$ and $\dfrac{\pi}{6} \leq \theta \leq \dfrac{\pi}{2}$ and $1\leq r\leq 2 \csc \theta$

Work Step by Step

Conversion of polar coordinates and Cartesian coordinates are as follows: a)$r^2=x^2+y^2 \implies r=\sqrt {x^2+y^2}$ b) $\tan \theta =\dfrac{y}{x} \implies \theta =\tan^{-1} (\dfrac{y}{x})$ c) $x=r \cos \theta$ d) $y=r \sin \theta$ As $y=r \cos \theta =1\implies r=\csc \theta$ Therefore, the region described in polar coordinates is: $0 \leq \theta \leq \dfrac{\pi}{6}$ and $1\leq r\leq 2 \sqrt 3 \sec \theta$ and $\dfrac{\pi}{6} \leq \theta \leq \dfrac{\pi}{2}$ and $1\leq r\leq 2 \csc \theta$
Update this answer!

You can help us out by revising, improving and updating this answer.

Update this answer

After you claim an answer you’ll have 24 hours to send in a draft. An editor will review the submission and either publish your submission or provide feedback.